1
$\begingroup$

Hi!

Let $a(x)\geq0$, $x\in R^d$ and $\int_{R^d} a(x) dx=1$. Then the operator $Af = a*f -f$ is bounded on the space of continuous functions on $R^d$ vanishing at infinity and it is a generator of a Markov semigroup, therefore, its spectrum will be only on the left half-plane (and bounded, of course).

If we consider now the Banach space of all bounded continuous functions on $R^d$, then $A$ will be bounded as well, however, what can we say about left half plane. Could it be shown that this operator is dissipative?

$\endgroup$

1 Answer 1

1
$\begingroup$

If it's dissipative on $C_0({\bf R}^d)$ then it should be dissipative on $C_b({\bf R}^d)$ too.

First check this in the case that $a$ has bounded support. The point is that if $a$ is supported on a ball of radius $r$ about the origin then $Af(x)$ only depends on values of $f(y)$ for $|x - y| < r$. So if $f \in C_b({\bf R}^d)$ and $x \in {\bf R}^d$ then for $R \gg r$ the function $$g(y) = \cases{\big(1 - \frac{|x - y|}{R}\big)f(y)&if $|x - y| < R$\cr0&if $|x - y| \geq R$\cr}$$ will satisfy $Ag(y) \approx (1 - \frac{|x-y|}{R})Af(y)$ for $|x - y| < R$. Thus $(\lambda I - A)g(y) \approx (1 - \frac{|x-y|}{R})(\lambda I - A)f(y)$ and the dissipation inequality for $g$ will imply approximately the same inequality for $f$ at $x$.

For general $a$, approximate by $a$ with bounded support. This part is easy.

$\endgroup$
4
  • $\begingroup$ Thank you. It's a bit strange, since $C_0(R^d)$ is not dense in $C_b(R^d)$ in $sup$-norm. $\endgroup$
    – user34763
    Jun 8, 2013 at 4:42
  • $\begingroup$ No, but I guess the point is that dissipativeness of $A$ is a local property, so we don't need uniform approximation, only uniform approximation on compact sets. $\endgroup$
    – Nik Weaver
    Jun 8, 2013 at 5:46
  • $\begingroup$ I used as dissipation inequality $\| (\lambda - A) f\|\geq \lambda \|f\|$. It is not local. You probably mean the equivalent inequality via functionals. But is is true that the corresponding functionals (from Hanh-Banach theorem) will be "the same" for $f$ and $g$? $\endgroup$
    – user34763
    Jun 8, 2013 at 6:42
  • $\begingroup$ "Local" in the sense that $(\lambda - A)f(x)$ only depends on values of $f$ near $x$, assuming $a$ has bounded support. $\endgroup$
    – Nik Weaver
    Jun 8, 2013 at 7:38

Your Answer

By clicking “Post Your Answer”, you agree to our terms of service and acknowledge you have read our privacy policy.

Not the answer you're looking for? Browse other questions tagged or ask your own question.